LSAT and Law School Admissions Forum

Get expert LSAT preparation and law school admissions advice from PowerScore Test Preparation.

 Administrator
PowerScore Staff
  • PowerScore Staff
  • Posts: 8917
  • Joined: Feb 02, 2011
|
#41669
Complete Question Explanation
(The complete setup for this game can be found here: lsat/viewtopic.php?t=4937)

The correct answer choice is (A)

Monday is only a problem for workers controlled by sequencing rules such as the fourth rule. Hence, P and S are a prime target for analysis in this question.

If S works on Monday, then from the fourth rule P would have to work on Monday. However, answer choice (A) states that J would work with S on Monday. As this is impossible, answer choice (A) cannot occur and is correct.
User avatar
 LauraWorkman
  • Posts: 2
  • Joined: Oct 05, 2022
|
#98815
I understand that rule #4 makes it so answer choice A cannot be true. But doesn't rule #1 make it so answer choice E also cannot be true?
 Adam Tyson
PowerScore Staff
  • PowerScore Staff
  • Posts: 5153
  • Joined: Apr 14, 2011
|
#98816
That rule tells us that "at least one" of the workers is a supervisor, LauraWorkman. Having two supervisors working together doesn't violate that rule! Don't confuse "at least" with "exactly."
User avatar
 LauraWorkman
  • Posts: 2
  • Joined: Oct 05, 2022
|
#98817
Thanks for the explanation and quick reply!

Get the most out of your LSAT Prep Plus subscription.

Analyze and track your performance with our Testing and Analytics Package.